+2 Daumen
406 Aufrufe

Aufgabe:

a) Berechnen Sie unter Verwendung der unten aufgeführten Hinweise \( \int \limits_{0}^{\infty} \exp \left(-t^{2}\right) d t \).
Hinweise zu a):

(i) Betrachten Sie die Funktion \( g: \mathbb{R}^{2} \rightarrow \mathbb{R} \) mit

\( g(x, y)=y \cdot \exp \left(-y^{2}\left(1+x^{2}\right)\right) \cdot 1_{[0, \infty) \times[0, \infty)}(x, y), \)

und integrieren Sie diese zuerst nach \( x \) und dann nach \( y \). Zeigen Sie dafur (mit Hilfe der Substitutionsregel) zunächst, dass

\( \int \limits_{0}^{\infty} y \cdot \exp \left(-x^{2} y^{2}\right) d x=\int \limits_{0}^{\infty} \exp \left(-t^{2}\right) d t \)

für beliebiges festes \( y>0 \), und verwenden Sie dieses Resultat.

(ii) Bestimmen Sie nun das obige Integral erneut, indem Sie jetzt zuerst nach \( y \) integrieren. Nutzen Sie dabei, dass

\( \frac{\partial}{\partial y}\left(-\frac{1}{2\left(1+x^{2}\right)} \cdot \exp \left(-y^{2}\left(1+x^{2}\right)\right)\right)=y \cdot \exp \left(-y^{2}\left(1+x^{2}\right)\right) \)

sowie

\( \frac{\partial}{\partial x} \arctan (x)=\frac{1}{1+x^{2}} \text {. } \)

(iii) Setzen Sie die beiden Ergebnisse gleich, und losen Sie nach dem gesuchten Integral auf. Berücksichtigen Sie dabei, welche Werte das Integral annehmen kann.


Problem:

Leider haben wir noch keine Integration im Mehrdimensionalem gemacht. Die Gleichung in (i) habe ich bewiesen. Wie kann man integrieren nach x und y? Macht das Unterschied was man zuerst integriert?

Avatar von

1 Antwort

+2 Daumen
 
Beste Antwort

Hi,
mit Hinweis (i) ist zu berechnen

\( \int_0^\infty \left[ \int_0^\infty y\cdot e^{-y^2\left(1+x^2\right)} dx \right] dy= \int_0^\infty e^{-y^2}\left[ \int_0^\infty y\cdot e^{-y^2x^2} dx \right] dy = \int_0^\infty e^{-y^2}\left[ \int_0^\infty y\cdot e^{-t^2}\cdot \frac{1}{y} dt \right] dy = \left( \int_0^\infty e^{-t^2}dt \right)^2 \)

Die vorletzte Gleichheit erhält man mit der Substitution \( t=xy \)

Mit Hinweis (ii) ist zu berechnen

\( \int_0^\infty \left[ \int_0^\infty y\cdot e^{-y^2\left(1+x^2\right)} dy \right] dx = \int_0^\infty \left[ -\frac{1}{2(1+x^2)}e^{-y^2(1+x^2)} \right]_0^\infty dx = \int_0^\infty\frac{1}{2(1+x^2)}dx = \frac{1}{2} \left[ \arctan(x) \right]_0^\infty = \frac{\pi}{4} \)

D.h. es gilt

\( \left( \int_0^\infty e^{-t^2}dt \right)^2 = \frac{\pi}{4} \)

Also

\( \left( \int_0^\infty e^{-t^2}dt \right) = \frac{\sqrt{\pi}}{2} \)

Avatar von 39 k

Ein anderes Problem?

Stell deine Frage

Willkommen bei der Mathelounge! Stell deine Frage einfach und kostenlos

x
Made by a lovely community